Q12

 
meyerkate17
Thanks Received: 0
Vinny Gambini
Vinny Gambini
 
Posts: 2
Joined: February 25th, 2012
 
 
 

Q12

by meyerkate17 Sat May 05, 2012 11:10 pm

Hi,

I'm having trouble with question 12. I understand how you got the diagram, but could you please walk me through this question?

Thanks!
 
timmydoeslsat
Thanks Received: 887
Atticus Finch
Atticus Finch
 
Posts: 1136
Joined: June 20th, 2011
 
This post thanked 3 times.
 
trophy
Most Thanked
trophy
First Responder
 

Re: Q12

by timmydoeslsat Sun May 06, 2012 8:47 pm

We are given a local question asking us what could be a pair of connected cities if P is connected with exactly 3 cities.

We are given a rule in our game of: PT ---> ~PV

That is a rule that tells us "not both." In other words, we cannot have Philadelphia be connected to both T and V, at least 1 will not be connected to P.

P can connect to any one of: HMTV

Well, with it being the case of T --->~V with P, we know that P must be connected with H, M, and one of T/V. This is where our other rule is triggered. If H is a connection, so must T. So P will be connected with HMT.

It would have been nice for the test writers to simply give us an answer choice of P and H/M/T. But they do not.

A) H and M = never can work.
B) H and V = defer judgment for now.
C) M and T = M must be with P.
D) M and V = M must be with P.
E) P and V = cannot work in this situation.

B wins by elimination.
 
syousif3
Thanks Received: 0
Jackie Chiles
Jackie Chiles
 
Posts: 36
Joined: July 19th, 2012
 
 
 

Re: Q12

by syousif3 Sat Aug 25, 2012 12:05 am

I dont think any one those answers could be right for one thing you just pointed out that V is not included so there is no way B can be true BUT it is the best out of all the other answers. This is the first time I see such an answer for LG.
 
soo.jee.lee.13
Thanks Received: 0
Vinny Gambini
Vinny Gambini
 
Posts: 1
Joined: July 29th, 2012
 
 
 

Re: Q12

by soo.jee.lee.13 Tue Sep 18, 2012 6:24 pm

syousif3 Wrote:I dont think any one those answers could be right for one thing you just pointed out that V is not included so there is no way B can be true BUT it is the best out of all the other answers. This is the first time I see such an answer for LG.


B (V connected with H) is entirely possible: while P is connected to T, H, and M, V could be connected to T and H without trouble. (In fact, V has to be connected to T at the very least, if not both.)

This game was pretty confusing, though! I'm going through the different questions one by one on this forum.